Patho 2 Exam 2-PrepU

Réussis tes devoirs et examens dès maintenant avec Quizwiz!

Chapter 30

--

A nurse suspects an older adult client is experiencing benign prostatic hyperplasia (BPH). Which question should the nurse ask this client? "Do you have any difficulty in achieving an erection?" "Have you noticed any unusual discharge from your penis?" "Has the force of your urinary stream decreased?" "Have you experienced a decrease in your sex drive?"

"Has the force of your urinary stream decreased?" Explanation: Because the prostate surrounds the urethra, enlargement of the gland can produce urinary obstruction.

A client is diagnosed with Addison disease. What statement by the client indicates an understanding of the discharge instructions by the nurse? "Once the symptoms go away, I will be able to stop taking my medication." "If I have surgery, it will cure me." "I should be able to control my condition with diet and exercise." "I will have to take my medication for the rest of my life."

"I will have to take my medication for the rest of my life." Explanation: Addison disease, like type I diabetes, is a chronic metabolic disorder that requires lifetime hormone replacement therapy. The daily regulation of the chronic phase of Addison disease is usually accomplished with oral replacement therapy, with higher doses being given during periods of stress.

The mother of a 2-year-old newly diagnosed with type 1 diabetes asks why insulin has to be given by injection. The best response by the nurse is: "Insulin is destroyed by the stomach contents and has to be administered by injection." "When your child gets old enough, you will not have to administer injections." "Insulin needs to go directly into the vein to work best." "Your child is not old enough to swallow the pills needed to treat her diabetes."

"Insulin is destroyed by the stomach contents and has to be administered by injection." Explanation: Insulin is destroyed by the gastrointestinal tract and needs to be administered via injection or inhalation. Type 1 diabetes is not treated with oral medications at this time. Insulin is administered subcutaneously rather than in the vein. The statement about the mother not having to give injections once the child is older does not address the mother's concerns.

The nurse has just completed teaching a client, newly diagnosed with type 1 diabetes, about the treatment options. Which response by the client leads the nurse to conclude that additional teaching is needed? "If I forget my insulin in the morning, I should take it as soon as I can to prevent hyperglycemia." "An exercise plan will be helpful for prevention of long-term complications." "So I can stop my insulin if I start an exercise program." "If I work on losing some weight, that might help prevent complications later."

"So I can stop my insulin if I start an exercise program." Explanation: Clients with type 1 diabetes require insulin therapy from the time of diagnosis. Weight loss and dietary management may be sufficient to control blood glucose levels. Treatments which involve medical nutrition therapy, exercise, and insulin will help prevent complications later on as the client ages.

A male client is diagnosed with an inguinal hernia. Which statement by the client indicates that the nurse's teaching about the hernia has been effective? "The hernia is a loop of bowel protruding through a weak spot in my abdominal muscles." "Most men my age develop hernias so there is no cause for concern." "I will need immediate surgery since the hernia is blocking blood supply to my intestines." "My hernia is a result of a congenital abnormality."

"The hernia is a loop of bowel protruding through a weak spot in my abdominal muscles." Explanation: An inguinal hernia or "rupture" is a protrusion of the parietal peritoneum and part of the intestine through an abnormal opening from the abdominal cavity. A loop of small bowel may become incarcerated in an inguinal hernia (strangulated hernia), in which case its lumen may become obstructed and its vascular supply compromised.

A client has just been diagnosed with esophageal cancer. Although the client was losing weight and fatigued most days, the client attributed it to aging and working. The health care provider recommends chemotherapy and irradiation. However, the cancer has already metastasized. The client asks the nurse what to expect if agreeing to the treatments. How should the nurse respond?

"The therapies may shrink the cancer in an effort to increase your likelihood of surviving the cancer."

An older adult female comes in to the clinic for her yearly gynecologic appointment and informs the nurse that she is having pain when having intercourse with her spouse. The client has been postmenopausal for at least 5 years. What is the nurse's best response? "This is caused by a decrease in estrogen levels, but there are things that you may be able to use to help." "That is really a shame since you won't be able to enjoy sex anymore." "The cause of this is an increase in progesterone levels and you will have to take hormone replacement to control this." "This is caused by the superficial layer of the endometrium shedding."

"This is caused by a decrease in estrogen levels, but there are things that you may be able to use to help." Explanation: Decreased estrogen stimulation after menopause causes the vaginal mucosa to become thin and dry, often resulting in dyspareunia, atrophic vaginitis, and occasional vaginal bleeding. Low-dose estrogen suppositories may be used and the client should be encouraged to use a water-soluble vaginal lubricant. This does not mean that the client will no longer be able to have —or enjoy—sex. It is not caused by increased progesterone levels.

The nurse is caring for a 48-year-old female client who asks, "How do you know when you have reached menopause?" Which statement is an appropriate response by the nurse? "You are considered to be menopausal when you have not menstruated for a full year and your hemoglobin level does not fluctuate." "You are considered to be menopausal when have not menstruated for 6 months and FSH levels fluctuate." "You are considered to be menopausal when you have not menstruated for a full year and the hormone that fluctuates with menstruation no longer fluctuates." "You are considered to be menopausal when you have not menstruated for 2 years."

"You are considered to be menopausal when you have not menstruated for a full year and the hormone that fluctuates with menstruation no longer fluctuates." Explanation: A woman who has not menstruated for a full year or who has a persistently elevated FSH level is considered menopausal.

Chapter 32:Disorders of Endocrine system

--

Chapter 33:Diabetes Mellitus and Metabolic Syndrome

--

Chapter 39: Disorders of the male genitourinary system

--

Chapter 40: Disorders of Female Genitourinary System

--

Which manifestation indicates a client is at risk for developing diabetes mellitus? 2 hour oral GTT 175 mg/dL (9.7 mmol/L) Fasting blood glucose level 75 mg/dL (4.16 mmol/L) Hemoglobin A1 5.0% (.05) Serum potassium 4.2 mEq/L (4.2 mmol/L)

2 hour oral GTT 175 mg/dL (9.7 mmol/L) Explanation: Laboratory values that are considered normal are hemoglobin A1C less than 6.5 percent, fasting plasma glucose of (FPG) less than 100 mg/dL or less than 140 mg/dL 2 hours after an oral glucose tolerance test (GTT). A hemoglobin A1C value that is greater than or equal to 6.5 percent; a fasting blood glucose greater than 126 mg/dL, or a blood glucose level greater than 200 mg/dL 2 hours after a glucose tolerance test (GTT) indicate diabetes mellitus. Values between these levels are considered to place clients at increased risk for diabetes mellitus. Potassium levels do not directly correlate with a diagnosis of diabetes mellitus.

The nurse is reviewing assessment data and determines which client is at highest risk for developing type 2 diabetes? A 45-year-old obese female with a sedentary lifestyle A 10-year-old male whose grandmother has type 2 diabetes A 40-year-old male who has liver disease due to hepatitis A 60-year-old female with a history of gestational diabetes

A 45-year-old obese female with a sedentary lifestyle Explanation: The person most at risk for developing type 2 diabetes is the 45-year-old obese female with a sedentary lifestyle. Other risk factors include family history, over age 40, and history of gestational diabetes. The 60-year-old would have developed it before age 60, if there were additional risk factors. Diabetes and metabolic syndrome are due to pancreatic problems, not liver problems.

Select the client at greatest risk for developing colorectal cancer. A 45-year-old female who takes four to six aspirin per week for arthritis A 64-year-old female whose mother had colorectal cancer A 40-year-old male with a history of peptic ulcer disease A 26-year-old male with a history of irritable bowel syndrome

A 64-year-old female whose mother had colorectal cancer Colorectal cancer peaks at 60 to 70 years of age, and fewer than 20% of cases occur before age 50. Its incidence is increased among persons with a family history of cancer, persons with Crohn disease or ulcerative colitis, and those with familial adenomatous polyposis of the colon. Aspirin or other NSAIDs may protect against colorectal cancer. IBS and peptic ulcers are not risk factors.

A 79-year-old woman reports a recent onset of "nearly constant heartburn." During the assessment interview, she states that she has "lots of aches and pains." She states that she is not on any prescription medications but often takes aspirin for pain. The nurse should suspect what diagnosis? Staphylococcal infection Acute gastritis Gastric cancer Helicobacter pylori infection

Acute gastritis Aspirin is often implicated in cases of acute gastritis. H. pylori, gastric cancer, and staphylococcus infections do not normally cause reflux (heartburn) and are not directly linked to aspirin use.

A female client with anorexia nervosa tells the nurse that she has been experiencing amenorrhea. The nurse is aware that this is a result of: Increased estrogen production that occurs in the body Increased gonadotropin-releasing hormone (GnRH) production Alteration of critical body fat-to-muscle ratio needed for menses to occur A secondary decrease in sexual activity

Alteration of critical body fat-to-muscle ratio needed for menses to occur Explanation: Female clients with amenorrhea related to anorexia nervosa have decreased adipose tissue, which is required for hormone production. Estrogen and GnRH production is decreased. Sexual activity is not related to amenorrhea.

When assessing the client with acute pancreatitis, which of these diagnostic tests—consistent with the disease— does the nurse anticipate will be altered? Amylase and lipase Creatine kinase The transaminases Glucose values

Amylase and lipase Explanation: Serum amylase and lipase are the laboratory markers most commonly used to establish a diagnosis of acute pancreatitis.

The nurse is explaining ovulation to a female client. The nurse explains that follicle-stimulating hormone (FSH) and luteinizing hormone (LH) are important hormones needed to ovulate and are secreted by which gland? Adrenal Anterior pituitary Posterior pituitary Thyroid

Anterior pituitary Explanation: After puberty, the anterior pituitary secretes follicle-stimulating hormone (FSH) and luteinizing hormone (LH).

A male client has experienced a forceful hit to the testes. The nurse expects that the sensory nerves that transmit pain impulses are associated with which nervous system? Autonomic nervous system Endocrine nervous system Circulatory nervous system Enteric nervous system

Autonomic nervous system Explanation: The testes are innervated by fibers from both divisions of the autonomic nervous system. Associated sensory nerves transmit pain impulses, resulting in excruciating pain, especially when the testes are hit forcibly.

Which complication of acromegaly can be life threatening? Bone overgrowth causes arthralgias Splayed teeth result in impaired chewing Cardiac structures increase in size Vertebral changes result in kyphosis

Cardiac structures increase in size Explanation: While all the complications can exist, it is the enlargement of the heart and accelerated atherosclerosis that may lead to an early death. The teeth become splayed, causing a disturbed bite and difficulty in chewing. Vertebral changes often lead to kyphosis, or hunchback. Bone overgrowth often leads to arthralgias and degenerative arthritis of the spine, hips, and knees. Virtually every organ of the body is increased in size.

An adult client with growth-hormone deficiency related to hypopituitarism has been taking replacement therapy for several months. The client informs the nurse that she is having pain in the hand and wrist almost constantly. What does the nurse understand is a common side effect of this therapy that seems to have affected this client? Metacarpal fractures Carpal tunnel syndrome Tenosynovitis Rheumatoid arthritis

Carpal tunnel syndrome Explanation: The most common side effects of GH treatment in adults with hypopituitarism are peripheral edema, arthralgias, myalgias, carpal tunnel syndrome, paresthesias, and decreased glucose tolerance.

All diseases have risk factors. What is the most significant environmental risk factor for pancreatic cancer? Water pollution Cigarette smoking Air pollution Heavy metal toxicity

Cigarette smoking Explanation: In pancreatic cancer, the most significant and reproducible environmental risk factor is cigarette smoking.

Crohn disease is recognized by sharply demarcated, granulomatous lesions that are surrounded by normal-appearing mucosal tissue. The nurse recognizes these lesions to be defined by which description?

Cobblestone A characteristic feature of Crohn disease is the sharply demarcated, granulomatous lesions that are surrounded by normal-appearing mucosal tissue. When there are multiple lesions, they are often referred to as "skip lesions" because they are interspersed between what appear to be normal segments of the bowel. The surface of the inflamed bowel usually has a characteristic "cobblestone" appearance resulting from the fissures and crevices that develop, surrounded by areas of submucosal edema.

The newborn nursery nurse is obtaining a blood sample to determine if a newborn has congenital hypothyroidism. What long-term complication is the nurse aware can occur if this test is not performed and the infant has congenital hypothyroidism? Dehydration from diarrhea Accelerated growth Cretinism Irritability and restlessness

Cretinism Explanation: Congenital hypothyroidism is a common cause of preventable intellectual disability. It affects approximately 1 in 4000 infants. The manifestations of untreated congenital hypothyroidism are referred to as cretinism. The term does not apply to the normally developing infant in whom replacement thyroid hormones therapy was instituted shortly after birth.

A nurse reading a sigmoidoscopy report notes that a client was found to have skip lesions. The nurse interprets this as an indication of:

Crohn disease Skip lesions, demarcated granulomatous lesions that are surrounded by normal-appearing mucosal tissue, are a characteristic feature of Crohn disease.

The nurse is caring for a client with liver disease who has edema throughout the body. When reviewing the medical record, the nurse recognizes that which altered diagnostic test is consistent with development of edema? Elevated ammonia Decreased albumin Decreased prothrombin Elevated cholesterol

Decreased albumin Explanation: Altered function of the liver causes decreased levels of plasma proteins, particularly albumin, which contributes to edema formation.

The male reproductive system is composed of two systems. The nurse knows that which system is responsible for the transportation and storage of sperm? Symphysis pubis Ductile system Accessory glands Pampiniform plexus

Ductile system Explanation: The ductile system (epididymis, vas deferens, and ejaculatory ducts) transports and stores sperm, and assists in their maturation. The accessory glands (seminal vesicles, prostate gland, and bulbourethral glands) prepare the sperm for ejaculation.

A client who has been diagnosed with premenstrual syndrome (PMS) asks if there is anything to help alleviate the symptoms. The best response would be: Decrease intake of fluids to reduce fluid retention. Increase intake of caffeine products during this period. Eat a diet low in simple sugars and high in protein. Limit activities and maintain bed rest as necessary.

Eat a diet low in simple sugars and high in protein. Explanation: Treatment for PMS includes treating various symptoms. A diet low in sugar and high in protein and reduction in caffeine intake are recommended. Other suggestions include diuretics to reduce fluid retention, continue fluid intake as usual, regular exercise, and analgesics for pain.

Pharmacologic treatment for peptic ulcers has changed over the past several decades. The nurse knows that the goal for pharmacologic treatment is focused on: Neutralizing blood count Increasing acid production Eradicating Helicobacter pylori (H. pylori) Promoting special diet

Eradicating Helicobacter pylori (H. pylori) Treatment of peptic ulcer is aimed at eradicating the cause and promoting a permanent cure for the disease. Pharmacologic treatment focuses on eradicating H. pylori, relieving ulcer symptoms, and healing the ulcer crater. Acid-neutralizing, acid-inhibiting drugs and mucosa-protective agents are used to relieve symptoms and promote healing of the ulcer crater. There is no evidence that special diets are beneficial in treating peptic ulcer.

Parents of a 20-month-old infant report that he refuses food or eats poorly and that he grimaces when he swallows. He also is irritable and cries a lot. The mother is worried that he ate something inappropriate this morning, because he vomited something that looked like coffee grounds. Which health problem would the care team first suspect?

Esophagitis from gastrointestinal reflux

A nurse on a medical unit is providing care for a 37-year-old female client who has a diagnosis of Graves disease. Which assessments should the nurse prioritize? Skin integrity and distribution of adipose tissue Eye health and visual acuity Signs and symptoms of decreased bone density Cognition and judgment

Eye health and visual acuity Explanation: The ophthalmopathy of Graves disease can cause severe eye problems, including tethering of the extraocular muscles resulting in diplopia; involvement of the optic nerve, with some visual loss; and corneal ulceration because the lids do not close over the protruding eyeball (due to the exophthalmos). Eye assessment is consequently a priority over assessment of skin integrity, cognition, or musculoskeletal status.

Good hand-washing techniques are important in health care. The nurse knows that bacterial infections can be prevented by good hand washing techniques. Which route of transmission is most common for Clostridium difficile? Sexual transmission Iatrogenic transmission Vertical transmission Fecal-oral transmission

Fecal-oral transmission After antibiotic therapy has made the bowel susceptible to infection, colonization by C. difficileoccurs by the oral-fecal route. C. difficile infection usually is acquired in the hospital, where the organism is most commonly encountered.

The fallopian tubes are narrow tubes that attach bilaterally to the uterus. Within the fallopian tube, fertilization of the ovum takes place. The end of the fallopian tube nearest the ovary is funnel-like. What are the fringed, finger-like projections around the funnel-shaped opening of the fallopian tube called? Fimbriae Oviducts Cilia Fallopian fingers

Fimbriae Explanation: The end of the fallopian tube nearest the ovary forms a funnel-like opening with fringed, finger-like projections called fimbriae, which pick up the ovum after its release into the peritoneal cavity after ovulation.

Which test can the nurse prepare the client for to determine the differentiation between a benign and malignant thyroid disease? Ultrasonography Fine-needle aspiration biopsy CT scan MRI

Fine-needle aspiration biopsy Explanation: Fine-needle aspiration biopsy of a thyroid nodule has proved to be the best method for differentiation of benign from malignant thyroid disease. Ultrasonography can be used to differentiate cystic from solid thyroid lesions, and CT and MRI scans are used to demonstrate tracheal compression or impingement on other neighboring structures.

A nurse is teaching a client diagnosed with Crohn disease about potential complications. The most appropriate information for the nurse to include would be: Chronic constipation Fistula formation Difficulty swallowing Excessive weight gain

Fistula formation Complications of Crohn disease include fistula formation, abdominal abscess formation, and intestinal obstruction. Clients with Crohn disease are at risk for weight loss and/or diarrhea. The disease does not cause difficulty swallowing as it typically is in the bowel.

A client has just become pregnant and there is a consequent increase in secretions from the columnar epithelium of her endocervix. What effect will this have? A sharp increase in the pH of the vaginal environment Inhibition of FSH synthesis Formation of a mucoid plug to protect her uterus Inhibition of ripening of multiple oocytes

Formation of a mucoid plug to protect her uterus Explanation: Secretions from the columnar epithelium of the endocervix protect the uterus from infection, alter receptivity to sperm, and form a mucoid "plug" during pregnancy. These secretions do not directly affect pH, FSH levels, or the function of oocytes.

A woman with a regular 28-day menstrual cycle wishes to become pregnant. At which point in the menstrual cycle is the optimal time for intercourse to end in fertilization of the ovum? Third day after menses ceases Fourteenth day after menses begins Fourth day after menses begins Twentieth day after menses ceases

Fourteenth day after menses begins Explanation: Ovulation usually occurs around the 14th day after menstruation begins. It is the best time to expect the ovum and sperm to join in fertilization.

A client is diagnosed with type 2 diabetes mellitus and begins to follow a nutritional plan at home. What result at the follow-up visit indicates a successful outcome?

Glycosylated hemoglobin 5.2% (0.52) Explanation: The goals of the nutritional plan for type 2 diabetes mellitus include normal glucose levels, normal lipid levels, weight loss to ideal body weight (or at least 5% to 10% of total body weight) and regulating blood pressure. High-density cholesterol should be above 60 mg/dL (1.55 mmol/L).

An 11-year-old boy has poorly controlled type 1 diabetes. In addition to assessing for complications of hyperglycemia, the nurse should assess for which developmental abnormality? Growth retardation Indications of glucocorticoid excess Exophthalmos Signs and symptoms of hyperthyroidism

Growth retardation Explanation: Poorly controlled diabetes in children has been shown to affect stature directly, even when growth hormone levels are normal. It is not linked to hyperthyroidism, exophthalmos, or Cushing syndrome.

The nurse is assessing a client with a collection of blood in the tunica vaginalis of the scrotum. How does the nurse correctly document this in the medical record? Hydrocele Varicocele Hematocele Spermatocele

Hematocele Explanation: Disorders of the scrotum and testes include collection of fluid (hydrocele), blood (hematocele), or sperm (spermatocele) in the tunica vaginalis; another disorder involves varicosities of the veins in the pampiniform venous plexus (varicocele).

A client tells the nurse that he is concerned about developing hepatitis after being exposed to contaminated feces, saliva, and food. The nurse is aware that the client is at risk for: Hepatitis C Hepatitis D Hepatitis B Hepatitis A

Hepatitis A Hepatitis A is normally transmitted through the fecal-oral route by drinking contaminated milk or water and eating shellfish from infected waters. Hepatitis B is transmitted through infected blood or serum. Hepatitis C is transmitted by recreational injection drug use. Hepatitis D occurs largely to persons at high risk for HBV infection.

A client is diagnosed with Crohn disease. The nurse instructs the client on which type of dietary needs?

High-calorie, vitamin, and protein diet Nutritional deficiencies are common in Crohn disease because of diarrhea, steatorrhea, and other malabsorption problems. A nutritious diet that is high in calories, vitamins, and proteins is recommended. Because fats often aggravate the diarrhea, it is recommended they be avoided. Elemental diets, which are nutritionally balanced but residue-free and bulk-free, may be given during the acute phase of the illness.

The nurse is performing a newborn assessment on a 2-day-old male infant. Assessment reveals the presence of the urethral opening on the ventral side of the penis. The nurse would document this finding as being: Epispadias Phimosis Balanitis Hypospadias

Hypospadias Explanation: In hypospadias, the termination of the urethra is on the ventral or underside surface of the penis. The etiology is unknown. Surgery is the treatment of choice and circumcision is avoided. Epispadias is a condition in which the urethral opening is on the dorsal or upper surface of the penis and is a less common condition. Phimosis refers to a tightening of the prepuce or penile foreskin that prevents its retraction over the glans. Balanitis is an acute or chronic inflammation of the glans penis, which generally occurs in 11% of adult males and 3% of boys.

The nurse is caring for the client with hepatocellular carcinoma. What does the nurse recognize is a cause of this disease? Exposure to nonsteroidal anti-inflammatory medications Gallbladder disease Diabetes Illness with hepatitis B or C

Illness with hepatitis B or C Explanation: With hepatitis B virus and hepatitis C virus, both of which become integrated into the host DNA, repeated cycles of cell death and regeneration afford the potential for development of cancer-producing mutations.

A client has developed priapism. For which outcome is this client at risk? Impotence Premature ejaculation Cancer of the penis Swelling of the testicles

Impotence Explanation: Priapism is an involuntary, prolonged (>4 hours), abnormal, and painful erection that continues beyond, or is unrelated to, sexual stimulation. The prolonged erection can result in ischemia and fibrosis of the erectile tissue with significant risk of subsequent impotence. The other results will not occur.

When the nurse is performing a health history for a client who is being admitted for hyperthyroidism, what symptom does the client report that the nurse would find associated with this disorder? Fatigue Weight gain Constipation Increase in appetite

Increase in appetite Explanation: Thyroid hormone enhances gastrointestinal function, causing an increase in motility and production of GI secretions that often results in diarrhea. An increase in appetite and food intake accompanies the higher metabolic rate that occurs with increased thyroid hormone levels. At the same time, weight loss occurs because of the increased use of calories.

When caring for the client with hepatic failure, the nurse recognizes that which problem places the client at increased risk for bleeding? Increased vitamin K Increased prothrombin time Decreased red blood cells Increased platelet count

Increased prothrombin time An increased prothrombin time would increase the risk for bleeding. Another factor that contributes to increased bleeding risk in clients with liver failure is malabsorption of vitamin K (decrease), which further impairs the synthesis of clotting factors. Increased platelets would not cause increased bleeding but rather increase the risk of clotting, while red blood cell count is not relevant.

Crohn disease is treated by several measures. Treatment with sulfasalazine will focus on which aspect of this disease? Increased appetite Inflammatory suppression Decreased bleeding tendency Immune suppression

Inflammatory suppression Treatment methods focus on terminating the inflammatory response and promoting healing, maintaining adequate nutrition, and preventing and treating complications. Several medications have been successful in suppressing the inflammatory reaction, including corticosteroids, sulfasalazine, metronidazole, azathioprine, 6-mercaptopurine, methotrexate, and infliximab.

Crohn disease not only affects adults but also can occur in children. The nurse assesses for which major manifestation in children with Crohn disease?

Malnutrition When Crohn disease occurs in children, one of the major manifestations may be retardation of growth and significant malnutrition.

As the sperm mature and are preparing for fertilization, what length of time are sperm able to survive in the male genital tract? Several days Several hours Many months Many weeks

Many weeks Explanation: A small quantity of sperm can be stored in the epididymis, but most are stored in the vas deferens or the ampulla of the vas deferens. The sperm can live for many weeks in the male genital tract; however, in the female genital tract, their life expectancy is 1 or 2 days.

A client is admitted to an acute care facility with a Clostridium difficile infection. The nurse anticipates administering which medication? Vancomycin Proton pump inhibitor Amoxicillin Metronidazole

Metronidazole Metronidazole is the medication of choice for treatment of C. difficile infections. It may be given intravenously or orally. Vancomycin is reserved for people who cannot tolerate metronidazole. Proton pump inhibitors are administered for prevention/treatment of ulcers. Amoxicillin and other antibiotic treatment is discontinued when a client is diagnosed with C. difficile.

A female client presents to her primary care physician with a report of very heavy bleeding during menstrual cycles. Tests indicate the condition is not from a hormone imbalance. Which intervention option is least invasive? Oral contraceptives Endometrial ablation Dilatation and curettage (D&C) Vaginal hysterectomy

Oral contraceptives Explanation: The least invasive treatment options for nonhormonal causes of excessive menstrual bleeding include oral contraceptives, cyclic progesterone therapy, or long-acting progesterone injection. Endometrial ablation is more invasive and eliminates the basement layer of the endometrium through use of heat, cold, microwaves, chemicals, or radiofrequency energy sources. D&C is a surgical scraping of the uterine lining. A hysterectomy, whether vaginal or abdominal, is the most invasive procedure to eliminate uterine bleeding.

The nurse on the urology unit teaches the nursing student that which test assists in detecting a recurrence of prostate cancer after prostatectomy? PSA level Human chorionic gonadotropin (HCG) level Alpha fetoprotein serum level White blood cell count

PSA level Explanation: A rising PSA (prostate-specific antigen) after treatment is consistent with progressive disease and is used to detect recurrence after total prostatectomy. Because the prostate is the source of PSA, levels should drop to zero after surgery.

When assessing a client with acute cholecystitis, the nurse anticipates the client's report of pain will be consistent with which description? Pain that starts as a diffuse ache and localizes over 24 to 48 hours Pain in the back, radiating to the groin Pain in the left lower quadrant, radiating to the back Pain in the right upper quadrant referred to the same shoulder

Pain in the right upper quadrant referred to the same shoulder Explanation: The pain of biliary colic begins abruptly and increases in intensity. It is usually located in the upper right quadrant or epigastric area and may be referred to the upper back, the right shoulder, or midscapular region.

The nurse caring for a client diagnosed with a ruptured appendix will assess for which high-risk complication? Diarrhea Vomiting Peritonitis Gastritis

Peritonitis Complications of a ruptured appendix include peritonitis, localized peril abscess formation, and septicemia.

An older adult client presents with a perforation of a peptic ulcer. The nurse will monitor for signs and symptoms of which problem? Peritonitis Vomiting Diarrhea Fecal impaction

Peritonitis Perforation occurs when an ulcer erodes through all the layers of the stomach or duodenum wall. With perforation, gastrointestinal contents enter the peritoneum and cause peritonitis.

Select the most common symptoms of diabetes. Select all that apply. Polyphagia Polycythemia Polydipsia Polyhydramnios Polyuria

Polydipsia Polyuria Polyphagia Explanation: The most commonly identified signs and symptoms of diabetes are often referred to as the three polys: (1) polyuria (i.e., excessive urination), (2) polydipsia (i.e., excessive thirst), and (3) polyphagia (i.e., excessive hunger). Polyhydramnios is a medical condition describing an excess of amniotic fluid in the amniotic sac. Polycythemia is a condition of increased red blood cells.

Which hormone is responsible for breast development and the increase in body temperature that occurs with ovulation? Prolactin Androgen Progesterone Testosterone

Progesterone Explanation: Progesterone is the hormone responsible for breast development and increase in body temperature during ovulation. The other options are not involved in either of these processes.

A nurse is evaluating diagnostic tests for a client suspected of having prostate cancer. Which result requires further evaluation? Prostate-specific antigen (PSA) of 12 ng/mL (12 µg/L) BUN 20 mg/dL (7.14 mmol/L) Uroflow examination 12 mL/second Creatinine 1.0 mg/dL (88.4 µmol/L)

Prostate-specific antigen (PSA) of 12 ng/mL (12 µg/L) Explanation: A prostate-specific antigen (PSA) level of 12 ng/mL is high (<4.0 ng/mL is considered normal); all other tests are within normal limits. Uroflow examination shows obstruction with a flow of less than 7 mL/second (average normal flow is 12 mL/second). The findings are positive for prostate cancer. PSA testing may be recommended annually for men, starting at age 50.

After several months of persistent heartburn, an adult client has been diagnosed with gastroesophageal reflux disease (GERD). Which treatment regimen is likely to be prescribed for this client's GERD? Surgical correction of the incompetent pylorus and limiting physical exercise Proton pump inhibitors; avoiding large meals; remaining upright after meals Anti-inflammatory medications; avoiding positions that exacerbate reflux; a soft-textured diet Weight loss and administration of calcium channel blocking medications

Proton pump inhibitors; avoiding large meals; remaining upright after meals Proton pump inhibitors block the final stage of gastric acid production, effectively controlling the root cause of the esophageal damage associated with GERD. The pylorus is not involved, and a soft diet is not indicated. Calcium channel blocking drugs would not address the problem.

The nurse is caring for a 34-year-old woman undergoing treatment for large endometrial lesions. What interventions should the nurse anticipate providing the client? Assisting with a Pap smear; administration of high-dose corticosteroids. Providing pain control; preparing the client for a laparoscopic procedure or hysterectomy. Administration of male androgens to minimize endometrial hyperplasia; facilitating a dilation and curettage. Administration of packed red blood cells to compensate for low hemoglobin; administering hormone therapy.

Providing pain control; preparing the client for a laparoscopic procedure or hysterectomy. Explanation: Pain control is central to treatment of endometriosis and surgical treatment is ideally performed laparoscopically, though hysterectomy is sometimes indicated. Bleeding is not a common symptom of endometriosis, and neither corticosteroids nor male androgens are common treatments.

A nurse is completing an abdominal assessment on a client suspected to have appendicitis. When the nurse applies and then releases pressure in the client's right lower quadrant, the client experiences tenderness. The nurse is documenting the presence of:

Rebound tenderness The nurse documents the presence of rebound tenderness, defined as tenderness that occurs when the nurse applies and then releases pressure to an area.

The nurse is performing physical assessment of the gastrointestinal tract. In which area does the nurse place the hands for palpation of the liver? Right upper quadrant Left upper quadrant Right lower quadrant Left upper quadrant

Right upper quadrant : The liver is the largest visceral organ in the body, located below the diaphragm in the right hypochondrium or right upper quadrant.

Following destruction of the pituitary gland, ACTH stimulation stops. Without ACTH to stimulate the adrenal glands, the adrenal glands' production of cortisol drops. This is an example of which type of endocrine disorder? Primary Secondary Tertiary Somatic

Secondary Explanation: In secondary disorders of endocrine function, the target gland is essentially normal, but defective levels of stimulating hormones or releasing factors from the pituitary system alter its function.

What intervention does the nurse suggest to a client experiencing pain associated with pancreatic cancer? Sleeping in a forward sitting position Applying moist, warm compresses to the back Assuming the supine position whenever possible Avoiding extremes in temperature change

Sleeping in a forward sitting position The most common pain associated with pancreatic cancer is a dull epigastric pain often accompanied by back pain, often worse in the supine position, and relieved by sitting forward. Avoiding temperature changes and the use of moist, warm compressions will afford little relief.

Which clinical manifestation would lead the nurse to suspect the client has malabsorption syndrome with a deficiency in fat absorption? Dry eyes Steatorrhea Glossitis Abdominal cramping

Steatorrhea In malabsorption syndrome, there is loss of fat in the stools and failure to absorb the fat-soluble vitamins. This can result in weight loss, steatorrhea, and fat-soluble vitamin deficiency. Cramping is associated with water/electrolyte imbalances. Eye problems like dry eyes is due to malabsorption of vitamin A. Glossitis is associated with folic acid deficiency.

A nurse administering a client's medication tells the client that a proton pump inhibitor has been added. When the client asks the purpose of the medication, the nurse responds that it is to prevent: Ulcerative colitis Crohn disease Malabsorption syndrome Stress ulcer

Stress ulcer Proton pump inhibitors are the first line of medications used in the prevention of stress ulcers.

A female client whose mother recently died from ovarian cancer asks, "Why the survival rate from ovarian cancer is so low?" Which response is most accurate? Ovarian cancer does not respond to radiation treatment. The virus that is linked to causing ovarian cancer is resistant to chemotherapy. It usually occurs in the older adult population, people who are not concerned with childbearing. Symptoms of ovarian cancer are nonspecific and therefore difficult to interpret.

Symptoms of ovarian cancer are nonspecific and therefore difficult to interpret. Explanation: Symptoms are often present and reported by women before diagnosis but are nonspecific and therefore difficult to interpret. Surgery and/or chemotherapy may be used to treat the disease. The incidence of ovarian cancer and mortality rate increase with age, with most cases occurring in women older than 50 years of age.

Hyperthyroidism that is inadequately treated can cause a life-threatening condition known as a thyroid storm. What are the manifestations of a thyroid storm? Select all that apply. Bradycardia Tachycardia Delirium Very high fever Very low fever

Tachycardia Delirium Very high fever Explanation: Thyroid storm is manifested by a very high fever, extreme cardiovascular effects (e.g., tachycardia, congestive failure, and angina), and severe CNS effects (e.g., agitation, restlessness, and delirium). The mortality rate is high. Very low fever and bradycardia are not manifestations of a thyroid storm.

A 14-year-old boy has been brought to the emergency department by his mother in excruciating pain that is radiating from his scrotum to his inguinal area. The boy's heart rate is 122 beats per minute and he has vomited twice before arrival at the hospital. Examination reveals that his scrotum is reddened and slightly swollen and the testes are firm to touch and tender, with extensive cremaster muscle contraction noted. What is the boy's most likely diagnosis? Hydrocele Varicocele Testicular torsion Epididymitis

Testicular torsion Explanation: The combination of the boy's age, signs, and symptoms is indicative of testicular torsion. Epididymitis normally lacks cremaster muscle involvement and hydrocele is marked by massive distention of the scrotum. Varicocele is often asymptomatic or marked by heaviness in the scrotum.

When explaining the role of liver Kupffer cells to a group of nursing students, which statement about the function of these cells is most accurate? These cells are the functional unit of the liver and are responsible for all liver secretions. The cells are capable of removing and phagocytizing old and defective blood cells. The primary function of Kupffer cells is to secrete bile. The role of the Kupffer cells is to provide at least 50% of cardiac output each minute to each lobular of the liver.

The cells are capable of removing and phagocytizing old and defective blood cells. Kupffer cells are reticuloendothelial cells that are capable of removing and phagocytizing old and defective blood cells, bacteria, and other foreign material from the portal blood as it flows through the sinusoid. This phagocytic action removes enteric bacilli and other harmful substances that filter into the blood from the intestine. Small tubular channels in the liver secrete bile. The functional unit of the liver is lobules. Approximately 25% of blood/min enters the liver through the hepatic artery.

A child is born with dwarfism to normal-sized parents. The physician is explaining how growth hormone (GH) plays a central role in the increase in stature that characterizes childhood and adolescence. What is the first step in the growth hormone chain of events? The hypothalamus secretes GHRH. The liver is stimulated. GH is released and circulates unbound in the plasma. Epiphyseal growth plates of long bones are influenced.

The hypothalamus secretes GHRH. Explanation: Like other pituitary functions, hypothalamic stimulation precedes hormone release. In the case of GH, stimulation is the result of GHRH by the hypothalamus. GH is then released by the pituitary gland, stimulating the liver to release IGFs, which ultimately causes the epiphyseal plates of long bones to grow.

A nursing student is studying pathophysiology as part of the nursing curriculum. Which statement made by the student indicates a need for further instruction? The male testes are located inside the abdominal cavity. The male testes are located outside the abdominal cavity. The testes are egg-shaped structures in the scrotum. The male testes are also known as gonads.

The male testes are located inside the abdominal cavity. Explanation: The male testes are located outside the abdominal cavity in the scrotum. The rest of the statements are true.

The pediatric nurse is teaching the mother of a newborn with hypospadias about this disorder. Which of these does the nurse convey? The foreskin of the penis is unable to be fully retracted. Medication will be given to correct this problem. The opening of the urethra is found on the underside of the penis. Undescended testes are frequently associated with hypospadias.

The opening of the urethra is found on the underside of the penis. Explanation: In hypospadias, which affects approximately 1 in 350 male infants, the termination of the urethra is on the ventral or underside surface of the penis.

A male client is being prepared for a physical examination of his prostate gland. How will the nurse explain the location of the prostate gland? The prostate gland is located in the inguinal canal. The prostate gland is located in the pelvis, inferior to the bladder. The prostate gland is located beneath the scrotal sac. The prostate gland is located in the meatus of the penis.

The prostate gland is located in the pelvis, inferior to the bladder. Explanation: The prostate is a fibromuscular and glandular organ lying just inferior to the bladder.

The community health nurse is teaching about prevention of hepatitis A. Which group does the nurse suggest will benefit from this vaccine? Those working for the Centers for Disease Control and Prevention Those traveling to Third World countries Homosexual women Those who have been recently exposed to hepatitis A

Those traveling to Third World countries Explanation: International hepatitis A vaccine is suggested for travelers to regions where sanitation is poor and endemic HAV infections are high, children living in communities with high rates of HAV infection, homosexually active men, and users of illicit drugs.

A client who has just undergone a thyroidectomy is experiencing high fever, tachycardia, and extreme restlessness. The nurse would interpret these findings as manifestations of which complication? Thyroid storm Hypothyroidism Myxedema coma Addisonian crisis

Thyroid storm Explanation: The symptoms this client is experiencing are related to thyroid storm and must be treated immediately to prevent death. Myxedema coma is related to hypothyroidism but typically does not occur after a thyroidectomy. Addisonian crisis is related to hypoadrenalism.

The nurse is caring for a baby with phimosis. Which of these does the nurse explain to the mother is the key feature of this disorder? A precancerous process that must be monitored closely Undescended testes Tightening of the foreskin preventing full retraction over the penis The bladder located outside the child's abdomen

Tightening of the foreskin preventing full retraction over the penis Explanation: Phimosis refers to a tightening of the prepuce or penile foreskin that prevents its retraction over the glans penis; as the child grows, retraction becomes possible in most cases.

A rare condition caused by gastrin-secreting tumors most commonly found in the small intestine or pancreas is called: Zollinger-Ellison syndrome Sickle cell anemia Creutzfeldt-Jakob disease Stevens-Johnson syndrome

Zollinger-Ellison syndrome Zollinger-Ellison syndrome is a rare condition caused by a gastrin-secreting tumor (gastrinoma).

The nurse is checking the pH of a client's vaginal tissue and finds the pH to be 4.3. The nurse interprets this finding as: normal. too acidic. neutral. too basic.

normal. Explanation: Vaginal tissue is usually moist, with a pH maintained within the bacteriostatic range of 4.0 to 4.5.

A client who has been diagnosed with acute symptomatic viral hepatitis is now in the icteric period. The nurse would expect the client to manifest: severe pruritus and liver tenderness. chills and fever. severe anorexia. disappearance of jaundice.

severe pruritus and liver tenderness. Explanation: Severe pruritus and liver tenderness are common during the icterus period. Chills, fever, and severe anorexia occur during the prodromal period. The disappearance of jaundice occurs in the convalescent phase.

Which treatment intervention is recommended for infants who have a severe case of hypospadias? splitting of the foreskin application of topical steroids surgical repair immediate circumcision

surgical repair Explanation: Surgical repair for normal sexual and urinary function to prevent psychological sequelae of having malformed genitalia is the most common treatment of hypospadias in the more severe cases. Circumcision is the treatment of choice for symptomatic phimosis. Splitting of the foreskin to allow for swelling and more blood flow is a radical procedure for prolonged erection. Application of topical steroids to suppress the overactive immune system is the treatment for balanoposthitis

The nurse is providing an education program to a group of men over the age of 18 years. An important topic the nurse is discussing relates to balanitis and its prevention. Which man does the nurse recognize is at high risk for this disorder? Select all that apply. A man with poor hygiene A man that has been circumcised A man that is on medication for hypertension A man with diabetes A man that is immunosuppressed

A man with poor hygiene A man that is immunosuppressed A man with diabetes Explanation: Balanitis is an acute or chronic inflammation of the glans penis, which generally occurs in 11% of adult males and 3% of boys. Males with poor hygiene, immunosuppression, or diabetes are more prone to balanitis. Males taking medications for hypertension and males that have been circumcised are at no greater risk for balanitis.

Treatment for diverticular disease includes increasing bulk in the diet to promote regular defecation. The nurse understands that increasing bulk will assist the colon to perform which function? Lowers the intraluminal pressure in the haustra Constricts the lumen of the colon Stimulates the colon to evacuate content quicker Increases the intraluminal pressure in the haustra

Lowers the intraluminal pressure in the haustra The combined contraction of the circular muscle and the lack of a continuous longitudinal muscle layer cause the intestine to bulge outward into pouches called haustra. Diverticula develop between the longitudinal muscle bands of the haustra in the area where the blood vessels pierce the circular muscle layer to bring blood to the mucosal layer. An increase in intraluminal pressure in the haustra provides the force for creating these herniations. The increase in pressure is thought to be related to the volume of the colonic contents. The scantier the contents, the more vigorous are the contractions and the greater is the pressure in the haustra. The increased bulk promotes regular defecation and increases colonic contents and colon diameter, thereby decreasing intraluminal pressure.

A 71-year-old male has been recently diagnosed with a stage III tumor of colorectal cancer, and is attempting to increase his knowledge base of his diagnosis. Which statement about colorectal cancer demonstrates a sound understanding of the disease?

"While diet is thought to play a role in the development of colorectal cancer, the ultimate causes are largely unknown." The etiology of cancer of the colon and rectum remains largely unidentified, though dietary factors are thought to exist. The prognosis, especially with stage III tumors, is poor. Simple and accurate screening tests do exist for colorectal cancer, while drugs are not implicated in the etiology.

Which pediatric assessment finding would the nurse recognize as an example of precocious puberty? A 13-year-old female with menarche and a recent growth spurt A 16-year-old male with genital enlargement and pubic hair growth A 7-year-old female with early menarche A 12-year-old male with advanced growth

A 7-year-old female with early menarche Explanation: The 7-year-old needs to be evaluated. Precocious puberty is defined as the appearance of secondary sexual development before the age of 8 years in girls. There is a shift toward a "new normal" of earlier ages of puberty, with black girls entering puberty earlier than white girls and children who are obese entering puberty at earlier ages than children of normal body weight for age. In boys of both races, the lower age limit remains 9 years; however, it is recognized that puberty can develop earlier in boys with obesity. The assessment findings of the 12-, 13-, and 16-year-olds are not of concern because they are normal findings.

While reviewing the role of glucagon regarding the regulation of blood glucose, the nurse knows that which situation could lead to an inhibition of glucagon release? Recent strenuous physical activity An increase in glucose levels A sharp decrease in blood glucose concentration Recent intake of large amounts of protein-rich food

An increase in glucose levels Explanation: Low blood sugar, intake of protein, and strenuous physical activity are associated with glucagon release. Lowered cellular metabolic needs and/or increased glucose levels would inhibit glucagon release.

When caring for the client with diabetic ketoacidosis, the nurse recognizes that fatty acids and ketones may be used for energy by most organs. Which organ does the nurse recognize is reliant on glucose as the major energy source? Heart Lungs Spleen Brain

Brain Explanation: Although many tissues and organ systems are able to use other forms of fuel, such as fatty acids and ketones, the brain and nervous system rely almost exclusively on glucose as a fuel source. Because the brain can neither synthesize nor store more than a few minutes' supply of glucose, normal cerebral function requires a continuous supply from the circulation.

A client with pancreatitis is admitted with weight loss, nausea, and vomiting. To maintain nutrition, the physician orders parenteral nutrition to be started. Knowing that a major side effect of parenteral nutrition is a hyperosmolar hyperglycemic state, the nurse should assess the client for which clinical manifestations? Irritability, bradycardia, wheezing noted on inspiration. Facial tics, shuffling gait, stiff joints. Dry lips, excess urine output, and seizures. Fever, chills, elevated BP of 170/101.

Dry lips, excess urine output, and seizures. Explanation: Hyperosmolar hyperglycemic state is characterized by high blood glucose (>600 mg/dL [33.3 mmol/L]), dehydration (dry lips), depression of sensorium, hemiparesis, seizures, and coma. The client may also experience weakness, polyuria, and excessive thirst. HHS may occur in various conditions, including type 2 diabetes, acute pancreatitis, severe infection, MI, and treatment with oral or parenteral nutrition solutions.

A client hospitalized with type 1 diabetes has been administered a scheduled dose of regular insulin. What are the primary actions of insulin? Select all that apply. Gluconeogenesis Protein synthesis Fat storage Glucose uptake by muscle and adipose tissue Promotion of glycogenolysis

Fat storage Glucose uptake by muscle and adipose tissue Protein synthesis Explanation: There are three actions of insulin: (1) it promotes glucose uptake by target cells and provides for glucose storage as glycogen; (2) it prevents fat and glycogen breakdown; and (3) it inhibits gluconeogenesis and increases protein synthesis. Glucagon, not insulin, promotes glycogenolysis

Type 1A diabetes is now considered an autoimmune disorder. What factors are considered necessary for type 1A diabetes to occur? Genetic predisposition, physiologic triggering event, allergic reaction to pancreatic alpha cells Diabetogenic gene from both parents, environmental triggering event, and a B-lymphocyte reaction to alpha cell antigens Diabetogenic gene from both parents, physiologic triggering event, and an allergic reaction to pancreatic delta cells Genetic predisposition, environmental triggering event, and a T-lymphocyte-mediated hypersensitivity reaction against some beta-cell antigen

Genetic predisposition, environmental triggering event, and a T-lymphocyte-mediated hypersensitivity reaction against some beta-cell antigen Explanation: Type 1A diabetes is thought to be an autoimmune disorder resulting from a genetic predisposition (i.e., diabetogenic genes); an environmental triggering event, such as an infection; and a T-lymphocyte-mediated hypersensitivity reaction against some beta-cell antigen.

The nurse is teaching a client diagnosed with Addison disease about the importance of lifetime oral replacement therapy. Which pharmacologic agent would be the drug of choice and included in this teaching plan? Ketoconazole Potassium supplements Insulin Hydrocortisone

Hydrocortisone Explanation: The daily regulation of the chronic phase of Addison disease is usually accomplished by oral replacement therapy, with higher doses being given during periods of stress. Hydrocortisone is usually the drug of choice in treating Addison disease. In mild cases, hydrocortisone alone may be adequate. Ketoconazole causes excessive breakdown of glucocorticoids and can also result in adrenal insufficiency. Clients with Addison disease usually have elevated potassium levels, and insulin is not a treatment for Addison disease.

A client comes to the clinic with fatigue and muscle weakness. The client also states she has been having diarrhea. The nurse observes the skin of the client has a bronze tone and when asked, the client says she has not had any sun exposure. The mucous membranes of the gums are bluish-black. When reviewing laboratory results from this client, what does the nurse anticipate seeing? Positive C-reactive protein Elevated WBC count Increased levels of ACTH Increase in sedimentation rate

Increased levels of ACTH Explanation: Hyperpigmentation results from elevated levels of ACTH. The skin looks bronzed or suntanned in exposed and unexposed areas, and the normal creases and pressure points tend to become especially dark. The gums and oral mucous membranes may become bluish-black. The amino acid sequence of ACTH is strikingly similar to that of melanocyte stimulating hormone; hyperpigmentation occurs in more than 90 percent of persons with Addison disease and is helpful in distinguishing the primary and secondary forms of adrenal insufficiency.

A client has been diagnosed with a Bartholin gland cyst. The nurse expects the client may experience which symptoms if this becomes infected? Pain, tenderness, and dyspareunia Pain, erythema, and dystonia Pain, dysphagia, and tenderness Distention, hyperthermia, and dyspnea

Pain, tenderness, and dyspareunia Explanation: A Bartholin gland cyst is a fluid-filled sac that results from occlusion of the duct system in the Bartholin gland. Acute symptoms are usually the result of infection and include pain, tenderness, and dyspareunia.

The nurse knows that which structures are a part of the male reproductive system? Select all that apply. Urinary bladder Prostate gland Pair of kidneys Pair of gonads Seminiferous tubules

Pair of gonads Seminiferous tubules Prostate gland Explanation: The male reproductive system consists of a pair of gonads (i.e., testes), a system of excretory ducts (i.e., seminiferous tubules and efferent ducts), the accessory organs (i.e., epididymis, seminal vesicles, prostate, and Cowper glands), and the penis.

The nurse is caring for a client with metastatic pancreatic cancer who is to undergo a surgical resection and radiation therapy. What does the nurse recognize is the goal of these interventions? Cure of the cancer Removal of the tumor Palliation of symptoms Diagnosis of the tumor

Palliation of symptoms Explanation: Surgical resection of the tumor is done when the tumor is localized, which occurs in 10% to 15% of cases. Most cancers of the pancreas have metastasized by the time of diagnosis. Surgical resection is reserved for palliative measures in most instances.

The nurse assessing a client with a diagnosis of cholelithiasis will look for pain in which area? Right upper quadrant Right lower quadrant Left lower quadrant Left upper quadrant

Right upper quadrant The pain is usually located in the upper right quadrant or epigastric area and may be referred to the upper back, right shoulder, or midscapular region. Typically, the pain is abrupt in onset, increases steadily in intensity, persists for 2 to 8 hours, and is followed by soreness in the upper right quadrant.

A client is managing diabetes with exercise and diet. The health care provider reviews the client's most recent laboratory results: fasting blood glucose level at 80 mg/dL (4.44 mmol/L) and a hemoglobin A1C of 5% (0.05). Select the response that best identifies the client. The client is at risk for developing hyperglycemia. The client is achieving normal glycemic control. The client needs to modify the diet related to the low readings. The client is at risk for an insulin reaction.

The client is achieving normal glycemic control. Explanation: The reading for the fasting blood glucose is appropriate, and the hemoglobin A1C level estimates good control of glucose levels over a 6- to 12-week period. This client should continue the exercise and diet routine as planned.

The nurse is educating a newly diagnosed client with Hashimoto thyroiditis who is to be discharged from the acute care facility. What should the nurse be sure to include in the education to prevent complications? The client should be informed about the signs and symptoms of severe hypothyroidism and the need for early intervention. The client should be informed that if he observes an increase in fatigue, an extra dose of his thyroid medication should be taken. The client should skip a dose of the thyroid supplement if he has symptoms of fever or restlessness. The client should not participate in any exercise activity that could utilize more thyroid hormone.

The client should be informed about the signs and symptoms of severe hypothyroidism and the need for early intervention. Explanation: Prevention is preferable to treatment and entails special attention to high-risk populations, such as women with a history of Hashimoto thyroiditis. These persons should be informed about the signs and symptoms of severe hypothyroidism and the need for early medical treatment.

A client with type 1 diabetes mellitus wishes to stop taking insulin injections. What option is appropriate? Increasing daily aerobic activity Using an insulin infusion pump Taking the herb chromium picolinate Taking metformin

Using an insulin infusion pump Explanation: Clients with type 1 diabetes mellitus require exogenous insulin because they have absolute lack of their own secretion. Insulin is a protein that would be digested if taken orally. To avoid injections, clients could use an insulin pump, which provides continuous infusion through a catheter placed under the skin that is changed every few days. While some herbal preparations may help lower blood glucose, they do not replace insulin. While aerobic activity helps mobilize glucose, it does not replace insulin. Metformin is an insulin sensitizer that enhances insulin utilization, but does not replace insulin.

A client has been diagnosed with alcohol-induced liver disease. He admits to the nurse, "I know what the lungs do, and I know what the heart does, but honestly, I have no idea what the liver does in the body." The nurse should tell the client that the liver: metabolizes most components of food and also cleans the blood of bacteria and drugs. is responsible for the absorption of most dietary nutrients, as well as the production of growth hormones. maintains a balanced level of electrolytes and pH in the body and stores glucose, minerals, and vitamins. contributes to the metabolism of ingested food and provides the fluids that the GI tract requires.

metabolizes most components of food and also cleans the blood of bacteria and drugs. Explanation: Protein, carbohydrate, and fat metabolism are performed by the liver. As well, it metabolizes drugs and removes bacteria by Kupffer cells. Absorption of nutrients takes place in the intestines. The liver does not produce the bulk of fluids secreted in the GI tract. The liver does not have a primary role in the maintenance of acid-base or electrolyte balance

A client with a history of cancer that metastasized to the liver has arrived at the outpatient clinic to have a paracentesis performed. The physician anticipates that the client will have more than 5 L of fluid removed. The physician has prescribed intravenous albumin following the procedure. The client asks why she needs "more fluids in my vein." The nurse responds: "Albumin works like your diuretics to help you get rid of excess fluid through your kidneys. It's just more potent than your home water pills." "Albumin is a volume expander. Since a lot of fluid was removed, you have a decrease in your vascular volume, so without this albumin, your kidneys will try to reabsorb and hold onto water." "After the albumin, your potassium level will stay steady and you should keep excess water weight off for several weeks." "Albumin will stay in your blood vessels a long time so that you will not seep out more fluid in your belly for at least a few weeks."

"Albumin is a volume expander. Since a lot of fluid was removed, you have a decrease in your vascular volume, so without this albumin, your kidneys will try to reabsorb and hold onto water." Large-volume paracentesis (removal of 5 L or more of ascitic fluid) may be done in persons with massive ascites and pulmonary compromise. Because the removal of fluid produces a decrease in vascular volume along with increased plasma renin activity and aldosterone-mediated sodium and water reabsorption by the kidneys, a volume expander such as albumin usually is administered to maintain the effective circulating volume.

A nurse educator is conducting a course for newly diagnosed diabetes clients. Which statement by a participant should the nurse follow up first? "I've had a little sore on the sole of my foot for a few days, but I'm sure it will eventually heal." "I know that we're supposed to get our eyes checked regularly, but my extended health insurance doesn't cover it." "I've started eating four or five smaller meals each day instead of three larger meals." "My doctor suggested checking my blood sugars twice a day, but I've started doing it before each meal and at bedtime."

"I've had a little sore on the sole of my foot for a few days, but I'm sure it will eventually heal." Explanation: Foot problems have been reported as the most common complication leading to hospitalization among people with diabetes. In people with diabetes, lesions of the feet represent the effects of neuropathy and vascular insufficiency and are a clinical priority. There is no particular risk associated with measuring glucose more often than normal. The client's dietary approach is not problematic. The nurse should facilitate the client's eye examinations but a new foot ulcer is a more pressing issue.

A client diagnosed with endometriosis asks the nurse how it is that she did not know about this when she had it for so long. What is the nurse's best response? "It is a rare disease." "There are no symptoms." "You needed to have surgery to diagnose it." "Its symptoms mimic those of other disorders "

"Its symptoms mimic those of other disorders " Explanation: Symptoms of endometriosis often mimic symptoms of other pelvic disorders and the severity of the symptoms does not always reflect the extent of the disease. There are no serum markers or non-invasive tests for the disease. It is not rare, and you do not need surgery to diagnose it; however, definitive diagnosis can be accomplished with laparoscopy and histological testing. Most cases are diagnosed on symptomatology.

After receiving change-of-shift report about the following four clients, which client should the nurse assess first? 53-year-old who has Addison disease and is due for a scheduled dose of hydrocortisone 31-year-old who has iatrogenic Cushing syndrome with a capillary blood glucose level of 204 mg/dL (11.32 mmol/L) 70-year-old returning from PACU following partial thyroidectomy who is extremely agitated, has an irregular pulse rate of 134, and an elevated temperature of 103.2°F (39.6°C) 22-year-old admitted with SIADH who has a serum sodium level of 130 mEq/L (130 mmol/L)

70-year-old returning from PACU following partial thyroidectomy who is extremely agitated, has an irregular pulse rate of 134, and an elevated temperature of 103.2°F (39.6°C) Explanation: Manipulation of a hyperactive thyroid gland during thyroidectomy can cause thyroid storm. It is manifested by very high fever, extreme cardiovascular effects (tachycardia, HF, angina), and severe CNS effects (agitation, restlessness, and delirium). The 22-year-old has normal sodium levels. The 31-year old has a high blood glucose level but not at a critical level. The medication schedule for the 53-year-old is lower priority. It is always preferred to give medications in timely manner; however, thyroid storms are the priority for this group of clients.

The health care provider is reviewing diurnal variation pattern in adrenocorticotropic (ACTH) levels. Select the typical diurnal variation pattern in adrenocorticotropic (ACTH) levels. ACTH increases in the morning and peaks again in the evening hours. ACTH maintains a consistent level regardless of the time of day. ACTH peaks in the morning and declines throughout the day. ACTH peaks in correspondence with food intake.

ACTH peaks in the morning and declines throughout the day. Explanation: ACTH levels have diurnal variation in which they reach their peak in the early morning (around 6 to 8 AM) and decline as the day progresses related to rhythmic activity of the CNS. The diurnal pattern is reversed in people who work during the night and sleep during the day. The rhythm also may be changed by physical and psychological stresses, endogenous depression, and liver disease or other conditions that affect cortisol metabolism.

The nurse is assessing a male client and finds abnormally large hands and feet, a bulbous nose, and a broad face with a protruding jaw. Based on these findings, which endocrine abnormality is most likely the cause for these physical changes? Acromegaly Hyperthyroidism Cushing syndrome Myxedema

Acromegaly Explanation: Enlargement of the small bones of the hands and feet and of the membranous bones of the face and skull results in a pronounced enlargement of the hands and feet, a broad and bulbous nose, a protruding jaw, and a slanting forehead. Bone overgrowth often leads to arthralgias and degenerative arthritis of the spine, hips, and knees. Virtually every organ of the body is increased in size. Enlargement of the heart and accelerated atherosclerosis may lead to an early death. Hyperthyroidism results from excess thyroid hormone. Myxedema and Cushing syndrome are the result of adrenal abnormalities and do not cause these bone changes.

Which nursing interventions would be considered priority when managing a client with life-threatening myxedema coma? Select all that apply. Administer 3% sodium IV solution to increase sodium levels. Administer 50% dextrose to raise glucose levels. Place on a warming bed to raise body temperature. Administer sedatives frequently to prevent seizures. Place on oxygen therapy to encourage deep breathing.

Administer 3% sodium IV solution to increase sodium levels. Administer 50% dextrose to raise glucose levels. Place on oxygen therapy to encourage deep breathing. Myxedema coma is a life-threatening, end-stage expression of hypothyroidism. It is characterized by coma, hypothermia, CV collapse, hypoventilation, and severe metabolic disorders that include low sodium, low glucose and lactic acidosis. Treatment includes aggressive management of precipitating factors; supportive therapy such as management of CV status, hyponatremia, and hypoglycemia; and thyroid replacement therapy. If hypothermia is present, active rewarming is contraindicated because it may induce vasodilation and vascular collapse. Administering sedatives frequently could be harmful since the person is unable to metabolize sedatives, analgesics, and anesthetic drugs.

The sister of a 25-year-old female client has breast cancer, and genetic testing showed her to have the BRCA1 gene. The client asks the nurse how she can prevent developing breast cancer herself. Which prophylactic interventions might be considered in this situation? Select all that apply. Breast magnetic resonance imaging (MRI) Bilateral mastectomy Bilateral oophorectomy Low-dose methotrexate Hormone replacement

Breast magnetic resonance imaging (MRI) Bilateral mastectomy Bilateral oophorectomy Explanation: If this client carries the BRCA1 gene like her sister, she has a 60% to 85% risk of developing breast cancer. Therefore, she should have a vigilant monitoring schedule for early breast cancer. MRI of the breast is preferred because of reduced radiation exposure and enhanced sensitivity. Prevention may also include bilateral mastectomy and/or bilateral oophorectomy to reduce hormones and the tissue that responds to them. The chemoprevention category that may be considered is aromatase inhibitors.

Which criterion about insulin would prompt a diagnosis of type 1 diabetes? Complete failure of insulin secretion Large amounts of insulin secreted Insulin not efficiently used Small amounts of insulin secreted

Complete failure of insulin secretion Explanation: In type 1 diabetes there is an absolute lack of insulin due to complete failure of the pancreas. In type 2 diabetes some insulin is produced but may not be properly used.

An older adult client expresses concern about straining to pass stools that are small and hard. What term will the health care provider use to document this in this client's chart? Dehydration Constipation Irritable bowel syndrome Diverticulosis

Constipation Constipation can be defined as the infrequent, incomplete, or difficult passage of stools. Diverticula are small pouches that bulge outward through the colon, or large intestine. Irritable bowel syndrome is a group of symptoms that occur together, including repeated pain in the abdomen and changes in bowel movements, which may be diarrhea, constipation, or both. Dehydration occurs when more water and fluids leave the body than enter it. Even low levels of dehydration can be the cause of headaches, lethargy, and constipation.

A 24-year-old woman undergoing a premarital screening test is found to have elevated levels of AST, ALT, and IgG, but no antibody-specific markers for viral hepatitis. A liver biopsy reveals inflammation and cellular damage. Which treatment is most likely to be effective for her? Corticosteroids and immunosuppressant drugs Interferon-alfa-2b Lamivudine Peginterferon and ribavirin

Corticosteroids and immunosuppressant drugs Explanation: This woman's hepatitis is probably caused by an autoimmune disorder rather than a virus. Lamivudine, peginterferon/ribavirin, and interferon alfa-2b are all antiviral agents. Autoimmune hepatitis is a chronic, usually progressive, inflammatory disease of the liver. Corticosteroids are the mainstay of treatment and have been shown to produce remission. Azathioprine, an immunosuppressant medication, is sometimes used along with prednisone. Using both medications may allow a client to take a smaller dose of prednisone, reducing its side effects. Most people with this disease require long-term maintenance treatment.

A client is diagnosed with adrenocorticotropic hormone deficiency (ACTH) and is to begin replacement therapy. Regarding which type of replacement will the nurse educate the client? Replacement therapy with prolactin Replacement therapy with synthetic thyroid hormone Growth hormone replacement therapy Cortisol replacement therapy

Cortisol replacement therapy Explanation: Cortisol replacement is started when ACTH deficiency is present; thyroid replacement when TSH deficiency is detected; and sex hormone replacement when LH and FSH are deficient. GH replacement is indicated for pediatric GH deficiency, and is increasingly being used to treat GH deficiency in adults.

A 28-year-old man presents with reports of diarrhea, fecal urgency, and weight loss. His stool is light-colored and malodorous, and it tends to float and be difficult to flush. He has also noted tender, red bumps on his shins and reports pain and stiffness in his elbows and knees. Sigmoidoscopy reveals discontinuous, granulomatous lesions; no blood is detected in his stool. Which diagnosis would his care team first suspect? Colon cancer Ulcerative colitis Diverticulitis Crohn disease

Crohn disease Crohn disease, like ulcerative colitis, causes diarrhea, fecal urgency, weight loss, and systemic symptoms such as erythema nodosum and arthritis. Unlike ulcerative colitis, it also causes steatorrhea but is not as likely to cause blood in the stool. The granulomatous "skip" lesions confirm the diagnosis of Crohn disease. Neither diverticulitis nor colon cancer would cause this combination of symptoms and signs.

A 23-year-old man is admitted to the hospital. He is experiencing polyphagia, polyuria, and polydipsia. He states that the condition has come on very suddenly. This client is likely to require what treatment? Sulfonylureas Meglitinides Biguanides Exogenous insulin injections

Exogenous insulin injections Explanation: Type 1 diabetes is the best diagnosis as the client has no other symptoms and the disease presented so rapidly. Insulin will likely be necessary and oral antihyperglycemics will not be sufficient. Meglitinides treat type 2 diabetes.

When comparing the symptomology of Crohn disease with that of ulcerative colitis, which symptoms are generally characteristic of only Crohn disease? Select all that apply. Perianal ulcers Toxic megacolon Risk for cancer of the colon Fistulas Bloody diarrhea

Fistulas Perianal ulcers Fistulas, strictures, and perianal ulcers are generally associated with Crohn disease, while bloody diarrhea, toxic megacolon, and the increased risk of colon cancer are associated only with ulcerative colitis.

The family of a client in the hospital with diabetes mellitus that is out of control asks the nurse to explain the client's recent weight loss while eating more than usual. How will the nurse respond? Lack of insulin raises circulating blood glucose levels. Fatty acids enter the glycolytic pathway to release energy. Surplus glucose is stored as glycogen in the liver. Glucose is unused without insulin, so body fats are used for energy.

Glucose is unused without insulin, so body fats are used for energy. Explanation: Most of these options are true statements, but they do not answer the question asked by the family. Normally, nutrients are metabolized in a number of ways. Glucose is transported into cells by insulin and then is broken down to carbon dioxide and water. When there is surplus glucose present, it is metabolized and stored as glycogen in the liver and skeletal muscles. Further surplus is converted by the liver to fatty acids and stored as triglycerides. When triglycerides are metabolized, the glycerol molecule enters the glycolytic pathway to release energy. Excess proteins are also converted to fatty acids for storage. Insulin is needed to transport glucose into cells, prevent fat breakdown, and inhibit gluconeogenesis. When diabetes is out of control there is lack of insulin. Weight loss occurs as the cells break down fats to use fatty acids for energy.

In the balance of secretions in the gastric mucosa by the parietal cells, which ion is produced to buffer the production of hydrochloric acid?

HCO3- Normally the secretion of hydrochloric acid by the parietal cells of the stomach is accompanied by secretion of bicarbonate ions (HCO3-), which protects the mucosa from injury, as long as they are produced in equal amounts.

The nurse is performing an assessment for a client who has hyperthyroidism that is untreated. When obtaining vital signs, what is the expected finding? Temperature 96°F (35.5°C) Heart rate 110 and bounding Respiratory rate 14 Blood pressure 180/110 mm Hg

Heart rate 110 and bounding Explanation: Cardiovascular and respiratory functions are strongly affected by thyroid function. With an increase in metabolism, there is a rise in oxygen consumption and production of metabolic end products, with an accompanying increase in vasodilation. Blood volume, cardiac output, and ventilation are all increased. Heart rate and cardiac contractility are enhanced as a means of maintaining the needed cardiac output. Blood pressure is likely to change little because the increase in vasodilation tends to offset the increase in cardiac output.

A 62-year-old man who is overweight has just been diagnosed with type 2 diabetes. The nurse educator is instructing him in the ways his diabetes can be controlled. The nurse should initially prioritize which action? Educating the client about the risks and management of hypoglycemia. Teaching the client about the action and safe administration of insulin. Helping the client make meaningful changes to his diet and activity level. Assisting the client with the appropriate choice of oral antihyperglycemics.

Helping the client make meaningful changes to his diet and activity level. Explanation: Weight loss and dietary management are the initial focus of treatment for type 2 diabetes. For many people with type 2 diabetes, the benefits of exercise include a decrease in body fat, better weight control, and improvement in insulin sensitivity. If good glycemic control cannot be achieved with exercise and diet, then antidiabetic agents and even insulin can be added to the treatment plan. Education is imperative, but there is no need to emphasize hypoglycemia, since the client will not be using insulin initially.

During periods of fasting and starvation, the glucocorticoid and other corticosteroid hormones are critical for survival because of their stimulation of gluconeogenesis by the liver. When the glucocorticoid hormones remain elevated for extended periods of time, what can occur? Adrenal hyperplasia Portal hypertension Hyperglycemia Hepatomegaly

Hyperglycemia Explanation: In predisposed persons, the prolonged elevation of glucocorticoid hormones can lead to hyperglycemia and the development of diabetes mellitus and starvation. They stimulate gluconeogenesis by the liver, sometimes producing a 6- to 10-fold increase in hepatic glucose production. A prolonged increase in glucocorticoid hormones does not cause hepatomegaly, portal hypertension, or adrenal hyperplasia.

A client comes to the clinic with fatigue and muscle weakness. The client also states she has been having diarrhea. The nurse observes the skin of the client has a bronze tone and when asked, the client says she has not had any sun exposure. The mucous membranes of the gums are bluish-black. When reviewing laboratory results from this client, what does the nurse anticipate seeing? Positive C-reactive protein Increase in sedimentation rate Increased levels of ACTH Elevated WBC count

Increased levels of ACTH Explanation: Hyperpigmentation results from elevated levels of ACTH. The skin looks bronzed or suntanned in exposed and unexposed areas, and the normal creases and pressure points tend to become especially dark. The gums and oral mucous membranes may become bluish-black. The amino acid sequence of ACTH is strikingly similar to that of melanocyte stimulating hormone; hyperpigmentation occurs in more than 90 percent of persons with Addison disease and is helpful in distinguishing the primary and secondary forms of adrenal insufficiency.

The nurse is reviewing laboratory results of a client who has liver failure. Which finding would place the client at increased risk for bleeding? Increased levels of vitamin K Increased platelet count Decreased number of red blood cells Increased prothrombin time

Increased prothrombin time Explanation: Clients with liver failure have malabsorption of vitamin K (decrease), which impairs the synthesis of clotting factors. An increased prothrombin time (a coagulation factor) places the client at risk for bleeding. Factors V, VII, IX, and X, prothrombin, and fibrinogen are synthesized by the liver; their decline in liver disease contribute to bleeding disorders. A decrease in RBC will not cause an increase in bleeding. An increased platelet count will cause the blood to clot.

The ovaries secrete both estrogen and progesterone. What is one function of progesterone in the body? Increases body temperature at ovulation Reduces levels of renin Causes moderate retention of sodium and water Enhances the coagulability of blood

Increases body temperature at ovulation Explanation: Although the mechanism is uncertain, progesterone increases basal body temperature and is responsible for the increase in body temperature that occurs with ovulation. The other answers are effects of estrogen.

A 40-year-old man who uses heroin intravenously was diagnosed with hepatitis C (HCV) 1 year ago and now has chronic viral hepatitis. Which statement by the client to his care provider would warrant correction by the nurse? Even though I'm sick, at least I won't feel sick most of the time. I know the medications to treat this aren't fantastic, but at least there are some options for controlling the virus. I'm not looking forward to all the side effects of the drug treatments for my HCV, but I hope I don't end up needing a liver transplant. It is at least a bit reassuring that my liver isn't undergoing damage when I'm not experiencing symptoms.

It is at least a bit reassuring that my liver isn't undergoing damage when I'm not experiencing symptoms. Liver damage persists both during symptomatic and asymptomatic periods of viral hepatitis C. Medications for treatment of the disease may cause unpleasant side effects. Liver transplant is a potential end-stage treatment option.

A client with severe hypoglycemia is unconscious. Which method of providing glucose should be avoided? Orange juice orally Glucose gel in the buccal pocket IM glucagon injection Dextrose IV

Orange juice orally Explanation: When clients are unconscious it is not safe to attempt to have them swallow liquids. Alternate routes that reduce the risk of choking such as buccal absorption, intramuscular or intravenous injections are preferred.

A woman in her 28th week of pregnancy tests positive for gestational diabetes mellitus and begins to follow a nutritional plan at home. What result at the follow-up visit indicates a successful outcome? Random blood glucose 85 mg/dL (4.72 mmol/L) Glycosylated hemoglobin 7.2% (.07) Gained 5 lb in one week 1% ketones present in urine

Random blood glucose 85 mg/dL (4.72 mmol/L) Explanation: The goals of the nutritional plan for gestational diabetes mellitus (GDM) include normal glucose levels, no ketosis, proper weight gain for the pregnancy, and adequate nutrition for fetal health.

A parent arrives in the endocrinology clinic with her 8-year-old son, concerned about his rapid development and tall stature. What significant assessment finding does the nurse recognize is important to report to the physician related to the development of precocious puberty? Significant genital enlargement The child is 20 lb over his target weight for height Enlarged head circumference Multiple dental caries

Significant genital enlargement Explanation: Diagnosis of precocious puberty is based on physical findings of early thelarche, adrenarche, and menarche. The most common sign in boys is early genital enlargement. Radiologic findings may indicate advanced bone age. People with precocious puberty are unusually tall for their age as children but short as adults because of the early closure of the epiphyses.

Inflammatory bowel disease (IBD) is used to designate two related inflammatory intestinal disorders: Crohn disease and ulcerative colitis. The nurse recognizes the difference between the distribution pattern between Crohn disease and ulcerative colitis. Which pattern describes Crohn's disease? Development of cancer Skip lesions Continuous involvement of the colon starting at the rectum Primarily rectum and colon involvement

Skip lesions Distribution patterns of disease manifest with skip lesions in Crohn disease and continuous involvement of the colon starting at the rectum in ulcerative colitis. Crohn disease primarily affects the ileum and secondarily the colon, and the development of cancer is uncommon. Ulcerative colitis primarily affects the rectum and left colon, and development of cancer is relatively common.

The nurse is providing discharge instructions for a client with Graves disease who has ophthalmopathy. What should the nurse be sure to include in the instructions to decrease exacerbation of this clinical manifestation? The client should be informed that if he begins to feel symptoms getting worse, he should take an extra dose of medication. The client should be strongly encouraged not to drink any alcohol. The client should be informed that he should not be in contact with other people during the acute phase. The client should be strongly urged not to smoke.

The client should be strongly urged not to smoke. Explanation: Ophthalmopathy can also be aggravated by smoking, which should be strongly discouraged. It is not necessary for the client to avoid contact with others. Alcohol is not contraindicated but should be limited when taking any medication regimen. The client should not adjust the doses of medications without first consulting the physician.

A newborn is screened for congenital hypothyroidism and is found to have the disorder. When educating the mother about the importance of the infant taking thyroid hormone supplement, what should be included in the education? The infant will have to take the medication only until he reaches puberty. The infant will have dosage levels adjusted as he grows. Once the dosage levels are adjusted, the infant will remain on the same amount throughout life. The medication dosage will increase every 6 months after laboratory testing.

The infant will have dosage levels adjusted as he grows. Explanation: Congenital hypothyroidism is treated by hormone replacement. Evidence indicates that it is important to normalize T4 levels as rapidly as possible because a delay is accompanied by poorer psychomotor and mental development. Dosage levels are adjusted as the child grows. When early and adequate treatment regimens are followed, the risk of intellectual disability in infants detected by screening programs is essentially nonexistent.

An obese adult has recently been diagnosed with type 2 diabetes. The nurse knows that the most likely treatment plan for this client will include which topics? Injectable insulin, nutrition management, and adequate hydration Continuous subcutaneous insulin infusion (CSII) and nutrition management Daily monitoring for urine ketones and weight loss measures. Weight loss, glucose monitoring, and oral antihyperglycemic medications

Weight loss, glucose monitoring, and oral antihyperglycemic medications Explanation: Persons with type 2 diabetes would be unlikely to require insulin initially and oral medications are likely to be of benefit as an addition to weight loss and glucose monitoring.

The obstetrical nurse is caring for a client who has been treated for gestational diabetes. When teaching the client about the causes of gestational diabetes, the nurse should include which risk factor in the teaching? Woman with a family history of diabetes First birth occurring during the teenage years Woman who has had a child under 5 lb (2.2 kg) First pregnancy

Woman with a family history of diabetes Explanation: Gestational diabetes occurs most commonly in black, Hispanic/Latino and Native American/First Nation women. It most frequently affects women with a family history of diabetes; a history of stillbirth or spontaneous abortion; women who previously gave birth to a newborn with fetal anomaly or had a previous large-for-gestational-age infant; those who are obese; those of advanced maternal age; or those who have had five or more pregnancies.

A client is admitted to the hospital with a suspected diagnosis of strangulated bowel. The nurse anticipates the client will need:

surgery to release the bowel. Strangulation and complete bowel obstruction require surgical intervention. Nasogastric tubes are used for adynamic obstructions that result from neurogenic or muscular impairment of peristalsis. Massage or diet would not relieve strangulation.

A client with history of alcohol abuse is brought to the emergency department after a weekend of heavy drinking, experiencing right upper quadrant pain, anorexia, nausea, jaundice and ascites. The nurse identifies these as manifestations of what disorder? Alcoholic hepatitis Fatty liver Cholestasis Cancer of the gallbladder

Alcoholic hepatitis Explanation: Fatty liver occurs when there is an accumulation of fat in the liver cells. The liver enlarges and becomes yellow. The fatty changes are reversible when alcohol intake stops. Alcoholic hepatitis is the next stage of liver disease after fatty liver. It is common when there is a sudden increase in alcohol intake and has a mortality rate of approximately 34 percent. The liver becomes inflamed and necrosis occurs. If the client survives and continues to use alcohol, alcoholic hepatitis develops into alcoholic cirrhosis.

A client has been admitted with diabetic ketoacidosis. The emergency department starts an IV to improve circulatory volume. If there is a sudden change in extracellular fluid osmolality which results in a too rapid blood glucose lowering, the nurse will likely observe which clinical manifestations? Increase urine output, thirst, vomiting. Rapid, deep respirations, palpitations, low BP. Chills, profuse sweating, weakness. Headaches, dizziness, change in level of consciousness.

Headaches, dizziness, change in level of consciousness. Explanation: A sudden change in osmolality of extracellular fluid can also occur when blood glucose levels are lowered too rapidly, and this can cause cerebral edema, more common in children than adults. The other responses all have some component of signs/symptoms of DKA rather than cerebral edema signs and symptoms.

Rotavirus is most severe in children under 24 months of age. What is a typical symptom of rotavirus infection? Mild to moderate fever that gets higher after the second day Vomiting that disappears around the second day but diarrhea continues Projectile vomiting that lasts for the course of the disease Fever that disappears after 7 days following antibiotic treatment

Vomiting that disappears around the second day but diarrhea continues Rotavirus infection typically begins after an incubation period of less than 24 hours, with mild to moderate fever, and vomiting, followed by onset of frequent watery, stools. The fever and vomiting usually disappear on about the second day, but the diarrhea continues for 5 to 7 days. Dehydration may develop rapidly, particularly in infants.

A female client with a strong family history of cardiovascular disease asks the nurse if she should take estrogen replacement while in menopause. What is the nurse's best response? "I wouldn't take it since it causes breast cancer." "Estrogen has additional cardioprotective actions and you and your physician should determine if this would benefit you." "If you don't take hormone replacement, you are at risk for myocardial infarction." "With your family history of cardiovascular disease, you should definitely take hormone replacement."

"Estrogen has additional cardioprotective actions and you and your physician should determine if this would benefit you." Explanation: Estrogens have additional cardioprotective actions, including direct antiatherosclerotic effects on the arterial wall, vasodilatation through endothelium-independent mechanisms, antioxidant activity, reduction of levels of angiotensin-converting enzyme and renin, reduction of homocysteine levels, improvement of peripheral glucose metabolism with subsequent decreased circulating insulin levels, and direct effects on cardiac function. The decision of whether hormone replacement therapy would be beneficial for this client should be made after consultation with the physician.

A 60 year-old man has long managed his type 1 diabetes effectively with a combination of vigilant blood sugar monitoring, subcutaneous insulin administration, and conscientious eating habits. This morning, however, his wife has noted that he appears pale and clammy and appears to be in a stupor, though he is responsive. She suspects that he has made an error in his insulin administration and that he is experiencing a hypoglycemic episode. Which action should be the wife's first choice? IV infusion of 50% dextrose and water solution. Administration of subcutaneous glucagon. Careful monitoring for level of consciousness and resolution of hypoglycemia. Administration of 15 to 20 g of glucose in a concentrated carbohydrate source.

Administration of 15 to 20 g of glucose in a concentrated carbohydrate source. Explanation: An insulin reaction necessitates intervention in addition to careful observation. The ideal response to an insulin reaction in a still conscious client is the administration of 15 to 20 g of glucose in a concentrated carbohydrate source. Glucagon or D50 would be indicated if the client is unconscious or unable to swallow.

When caring for the client with acute pancreatitis, which alterations does the nurse recognize is consistent with the disease? Hyperglycemia Leukopenia Hypertension Polyuria

Hyperglycemia Explanation: Serum amylase and lipase are the laboratory markers most commonly used to establish a diagnosis of acute pancreatitis. The white blood cell count may be increased, and hyperglycemia and an elevated serum bilirubin level may be present.

The nurse is caring for the client with pancreatic cancer. The nurse monitors the client for which complication? Hyperglycemia due to inability to synthesize insulin Bleeding related to lack of clotting factors Nutritional imbalance due to inability to synthesize protein Gallstones related to inability to digest fat

Hyperglycemia due to inability to synthesize insulin : The endocrine pancreas supplies the insulin needed to lower glucose levels in the blood; damage to the pancreas may alter this function, causing hyperglycemia.

A 30-year-old client who manages type 2 diabetes with glyburide presents at the emergency room reporting headache, confusion, and tachycardia. The client has come from a party at which the client drank two beers to celebrate running a half-marathon. Which is likely to be the cause of this client's? Hypoglycemia Somogyi effect Hyperosmolar hyperglycemic state Diabetic ketoacidosis

Hypoglycemia Explanation: In hypoglycemia, headache, difficulty in problem solving, disturbed or altered behavior, coma, and seizures may occur. At the onset, activation of the parasympathetic nervous system often causes hunger, and the initial parasympathetic response is followed by activation of the sympathetic nervous system; this causes anxiety, tachycardia, sweating, and constriction of the skin vessels (i.e., the skin is cool and clammy). In diabetic ketoacidosis, the client typically has a history of one or two days of polyuria, polydipsia, nausea, vomiting, and marked fatigue. Abdominal pain and tenderness may be experienced without abdominal disease, and the breath has a characteristic fruity smell. The most prominent manifestations of hyperosmolar hyperglycemic state are dehydration and neurologic signs including grand mal seizures, hemiparesis, Babinski reflexes, aphasia, muscle fasciculations, hyperthermia, hemianopia, nystagmus, and visual hallucinations; the client will also experience excessive thirst. The Somogyi effect describes a cycle of insulin-induced posthypoglycemic hyperglycemic episodes.

A nurse practitioner is providing care for a male client with a longstanding hiatal hernia. Which statement most accurately captures an aspect of the pathophysiology of hiatal hernias? If esophageal acid clearance is impaired, esophagitis can result. An incompetent pyloric sphincter and high fat diet are commonly implicated in the development of hiatal hernias. The root causes of hiatal hernias are normally treatable with medication. Paraesophageal hiatal hernias are common and are normally not treated if the client is asymptomatic.

If esophageal acid clearance is impaired, esophagitis can result. Erosive esophagitis can be a complication of hiatal hernias if esophageal acid clearance is significantly impaired. Paraesophageal hiatal hernias are more serious than the sliding variety and require treatment. The root cause of hiatal hernias—herniation of the stomach through the diaphragm—is not normally amenable to treatment with medication. The pyloric sphincter is not associated with hiatal hernias.

A 52-year-old male diagnosed with erectile dysfunction (ED) asks the nurse what will be implemented as a first option for treatment. The best response would be: Catheterization and increased fluids to increase vascular flow Surgical placement of a penile prosthesis Intravenous administration of a PDE-5 inhibitor Psychosexual counseling and PDE-5 inhibitor medications

Psychosexual counseling and PDE-5 inhibitor medications Explanation: Treatment methods include psychosexual counseling, androgen replacement therapy, oral and intravenous drug therapy, vacuum restriction devices, and surgery. Commonly prescribed drugs for ED are the selective inhibitors of phosphodiesterase-type 5 (PDE-5). The PDE-5 inhibitors are taken orally. Surgery would not be the initial therapy, and increasing fluids does not increase blood flow to the penis.

A nurse is performing a genital examination on a male client who develops an erection. What is the nurse's most appropriate intervention? Reassure the client that this is a normal response and continue with the examination Stop examining the client and leave the room Ignore the erection and continue with the examination Ask the client if he would prefer someone else to perform the examination

Reassure the client that this is a normal response and continue with the examination Explanation: Emission and ejaculation, which constitute the culmination of the male sexual act, are a function of the sympathetic nervous system. Genital stimulation can produce erection. In the flaccid or detumescent state, sympathetic discharge through alpha-adrenergic receptors maintains contraction of the arteries that supply the penis and vascular sinuses of the corpora cavernosa and corpus spongiosum. Parasympathetic stimulation produces erection by inhibiting sympathetic neurons that cause detumescence and by stimulating the release of nitric oxide to effect a rapid relaxation of the smooth muscle in the sinusoidal spaces of the corpus cavernosum.

A client is diagnosed with choledocholithiasis and acute suppurative cholangitis and is being rushed to surgery to prevent which possible complication? Impaired nutritional absorption Neurologic damage that can be caused by toxic bilirubinemia. Hepatitis that can result from the distention of the bile duct. The accumulation of purulent bile, which can cause sepsis.

The accumulation of purulent bile, which can cause sepsis. Explanation: Complications of choledocholithiasis include acute suppurative cholangitis when purulent bile fills and distends bile ducts. It is characterized by the presence of an altered sensorium, lethargy, and septic shock. Bilirubinuria and malabsorption are not emergency situations. This situation is not part of the etiology of hepatitis.

The nurse is caring for a client with viral hepatitis who has general malaise, easy fatigability, arthralgia, and anorexia. These manifestations correspond with what stage of the disease? Prodromal period Convalescent phase Chronic period Icterus phase

The nurse is caring for a client with viral hepatitis who has general malaise, easy fatigability, arthralgia, and anorexia. These manifestations correspond with what stage of the disease? You Selected: Prodromal period Correct response: Prodromal period Explanation: Acute symptomatic viral hepatitis can be divided into three phases: the prodromal period, icterus phase, and the convalescent phase. The prodromal period includes general malaise, myalgia, arthralgia, easy fatigability, and severe anorexia. There may also be nausea, vomiting, and diarrhea with mild, right-sided abdominal pain. The icterus phase usually follows the first phase by 5 to 10 days. Jaundice may accompany a temporary worsening of the initial symptoms. In the convalescent phase the client begins to feel better, regains the appetite, and jaundice subsides. The acute illness subsides gradually over 2 to 3 weeks.


Ensembles d'études connexes

Intro to Business chapter 5 and 6

View Set

3. Different Types of Neural cells

View Set

COMM4308 (Intercultural Comm) Quizzes

View Set

Becoming a World Power, 1872-1917, American History Chapter 5 Lesson 1 and 2

View Set

Pollution Responder 5.0 Recommended Enforcement Actions *

View Set

Chapter 1: Evolution and the Foundations of Biology

View Set

CTU Intro to Marketing: What is Marketing?

View Set